Chapter 33 Care of Patients with Infectious Respiratory Problems

Réussis tes devoirs et examens dès maintenant avec Quizwiz!

Rhinitis

-In both acute & chronic allergic rhinitis, the presence of the allergen causes a release of histamine & other mediators from WBC in the nasal mucosa. -Manifestations include- headache, nasal irritation, sneezing, nasal congestion, rhinorrhea (watery drainage from the nose), & itchy, watery eyes. -Management- focuses on symptom relief & patient education. -Drug therapy- antihistamines & decongestants- antibiotics only when a bacterial infection accompanies rhinitis.

if teeth are present, the patient on a ventilator should have their teeth brushed every ____

12 hrs

When does a patient with PTA usually improve

36 hours

When X consecutive resulta of sputum cultures are negative.....

3; the patient is no longer infectious

How long do you have to take TB drugs

6-12 months so noncompliance is an issue.

When taking an oral prophylaxis for anthrax, treatment must continue for...

60 days or longer depending on exposure

How long does acute tonsillitis usually last

7 to 10 days

ANS: A, B, C CBC, throat culture, and monospot testing can help to determine the causes of sore throat and fever. A biopsy is not needed. Human immune deficiency virus (HIV) testing would not be indicated unless the symptoms were a recurrent problem. Arterial blood gases would not be performed unless the client had dyspnea and a low oxygen saturation reading.

A client enters the clinic with an acute sore throat and a temperature of 101.5° F (38.5° C). What diagnostic testing does the nurse educate the client about? (Select all that apply.) a. Complete blood count (CBC) b. Throat culture c. Monospot test d. Arterial blood gas e. Biopsy f. HIV testing

Ethambutol

A client is taking INH, rifampin, pyrazinamide, and ethambutol for tuberculosis. The client calls to report visual changes, including blurred vision and reduced visual fields. Which medication may be causing these changes? A. Ethambutol B. INH C. Pyrazinamide D. Rifampin

Pulmonary empyema

A collection of pus in the pleural space.

Antihistamines & decongestants should be used with caution

An 80 year old man has a diagnosis of chronic rhinitis. Which statement is relevant to the care of this patient?

-Discontinue the use of the current nose drops or sprays

An adult patient diagnosed with rhinitis medicamentosa reports chronic nasal congestion. What does the nurse instruct the patient to do?

Place the patient on droplet precautions for the first 2 to 3 days.

An older adult pt residing in a long-term care facility demonstrates new onset of coughing & sneezing & rhinorrhea after his grandchildren came to visit him. He denies pain or fever. Which infection control procedures does the nurse instruct the LPN to initiate in order to protect the other residents?

Pulmonary Empyema

Collection of pus in pleural space Most common cause—pulmonary infection, lung abscess, infected pleural effusion Interventions: Empty empyema cavity Re-expand lung Control infection

Pneumonia

Excess fluid in lungs resulting from inflammatory process Inflammation triggered by infectious organisms, inhalation of irritants Community-acquired infectious pneumonia (CAP) Nosocomial or hospital-acquired (HAP) More resistant to some antibiotics than CAP

Which symptom of pneumonia may present differently in the older adult than in the younger adult? A. Crackles on auscultation B. Fever C. Headache D. Wheezing

Fever

What labs are drawn to detect polycythemia?

Hgb and Hct; electrolytes as they can be affected by acidosis

Pulmonary TB

Highly communicable; caused by Mycobacterium tuberculosis Transmitted via aerosolization Secondary TB (reactivation of the disease) Incidence increase related to onset of HIV

Where is an initial TB infection seen?

Initial infection is seen more often in the middle or lower lobes of the lung. -The local lymph nodes are infected & enlarged. An asymptomatic period usually follows the primary infection & can last for years or decades before clinical symptoms develop. *An infected person is not infectious to others until manifestations of disease occur.

Interventions for Lung Abcess

Interventions: Antibiotics Drainage of abscess Frequent mouth care for Candida albicans

Rhinitis interventions

Interventions: Drug therapy—antihistamines, leukotriene inhibitors, mast cell stabilizers, decongestants, antipyretics, antibiotics Complementary and alternative therapy—vitamin C, zinc Supportive therapy Assess for infection

Lung Abcess

Localized area of lung destruction Caused by liquefaction necrosis, usually related to pyogenic bacteria Pleuritic chest pain Patients usually have Hx of pneumonia, aspiration, or obstruction more aspiration causes: decreased ability to swallow, alcoholic blackouts (oops) and seizures.

The nurse plans discharge teaching for the client who is recovering from pneumonia. When is the best time to accomplish this? A. After the client has had his bath B. In the evening C. Midmorning or midafternoon D. When visitors are present

Midmorning or midafternoon

Manifestations of sinusitis

Nasal swelling and congestion, headache, facial pressure, and pain (usually worse when the head is tilted forward or is in a dependent position). Other manifestations include tenderness to touch over the involved area, low-grade fever, cough, and purulent or bloody nasal drainage.

Sinusitis nonsurgical management

Nonsurgical management: Broad-spectrum antibiotics Analgesics (pain and fever) Decongestants Steam humidification Hot/wet packs over sinus area Nasal saline irrigations Increased fluids If not effective evaluate sinus x-rays/CT scan

Assessment for Lung Abcess

PLEURITIC CHEST PAIN Often: febrile, pale, fatigued, cachectic

What is the treatment for PTA

Percutaneous needle aspiration of abscess Antibiotics Teach comfort measures

A client who has recently traveled to Vietnam comes to the emergency department with fatigue, lethargy, night sweats, and a low-grade fever. What is the nurse's first action? A. Contacts the health care provider for tuberculosis (TB) medications B. Performs a TB skin test C. Places a respiratory mask on the client D. Tests all family members for TB

Places a respiratory mask on the client

Community health nurses are tasked with providing education on prevention of respiratory infection for diseases such as the flu. Which target audience is given the highest priority? A. Homeless people B. Hospital staff C. Politicians D. Prison staff and inmates

Prison staff and inmates

Clinical manifestitions of TB

Progressive fatigue Lethargy Nausea Anorexia Weight loss Irregular menses Low-grade fever, night sweats Cough, mucopurulent sputum, blood streaks

Complications of strep

Rheumatic fever Acute glomerulonephritis Peritonsillar abcess Retropharyngeal abcess OTITIS MEDIA Sinusitis Mastoiditis Bronchitis Scarlet Fever

Rapid Antigen Test (RAT)

Screening process for group A streptococcal antigen is a priority intervention Results available in <15 min False negatives can occur The health care provider may need to scrape the back of the throat with the swab several times. This helps improve the chances of detecting bacteria. (Correct technique - - p. 643)

What is secondary TB?

Secondary TB is a reactivation of the disease in a previously infected person. It is more likely when defenses are lowered, such as with older adults and people with HIV disease. The upper lobes are the most common site of reactivation and are referred to as Simon's foci. -People who are at greatest risk for development of TB: Constant, frequent contact with an untreated person Those who have decreased immune function or HIV People who live in crowded areas such as long-term care facilities, prisons, & mental health facilities Older homeless people Abusers of injection drugs or alcohol Lower socioeconomic groups Foreign immigrants

Classifications of TB

TABLE 33-5 American Lung Association Classification 0 No TB exposure, not infected 1 TB exposure, no evidence of infection 2 TB infection, no disease 3 TB clinically active (patients with completed diagnostic evidence of TB—both a significant reaction to TB skin test and clinical or x-ray evidence of TB) 4 TB: not clinically active (patients with history of TB or abnormal chest x-ray film but no significant TB skin test reaction or clinical evidence) 5 TB: suspect (diagnosis pending); used during diagnostic testing of suspect patients for no longer than 3 months

Airborne

TB is transmitted via the ___________________ route.

Isoniazid (INH)

Teach the pt not to take medications such as Maalox with this medication Avoid drinking alcoholic beverages Take a multivitamin with B-complex Teach the patient to take in the morning, preferably before eating unless it causes an "upset stomach" Teach to take with food to decrease GI upset

Rifampin

Teach women that this drug reduced the effectiveness of oral contraceptives & that another form of birth control should be used while on this therapy. Teach the patient that urine will be orange in color. Avoid drinking alcoholic beverages

Keeping the head of the bed elevated 30 to 45 degrees

The RN is caring for a client with severe acute respiratory syndrome (SARS) who is receiving mechanical ventilation. Which nursing action should the nurse delegate to a nursing assistant? A. Keeping the head of the bed elevated 30 to 45 degrees B. Monitoring the impact of the ventilator on the client's oxygenation C. Performing oral care with disinfecting rinses every 12 hours D. Suctioning the client's endotracheal tube as needed

Not taking the medication could lead to an infection that is difficult to treat or to total drug resistance. The medications may cause nausea. The client should take them at bedtime.

The client has recently been released from prison and has just tested positive for tuberculosis (TB). What teaching points does the community health nurse want to stress for this client regarding medications? Select all that apply. A. Not taking the medication could lead to an infection that is difficult to treat or to total drug resistance. B. The medications may cause nausea. The client should take them at bedtime. C. The client is generally not contagious after 2 to 3 consecutive weeks of treatment. D. These medications must be taken for 2 years. E. These medications may cause renal failure.

"You will not be contagious to the people you have been living with."

The client is being discharged home with active tuberculosis. Which information does the nurse include in the discharge teaching plan? A. "You are not contagious unless you stop taking your medication." B. "You will not be contagious to the people you have been living with." C. "You will have to take these medications for at least 1 year." D. "Your sputum may turn a rust color as your condition gets better."

Arrange for a health care worker to watch the client take the medication.

The community health nurse is planning tuberculosis treatment for a client who is homeless and heroin addicted. Which action will be most effective in ensuring that the client completes treatment? A. Arrange for a health care worker to watch the client take the medication. B. Give the client written instructions about how to take prescribed medications. C. Have the client repeat medication names and side effects. D. Instruct the client about the possible consequences of nonadherence.

Client with possible pulmonary tuberculosis who currently has hemoptysis

The medical-surgical unit has one negative airflow room. Which of these four clients who have just arrived on the unit should the charge nurse admit to this room? A. Client with bacterial pneumonia and a cough productive of green sputum B. Client with neutropenia and pneumonia caused by Candida albicans C. Client with possible pulmonary tuberculosis who currently has hemoptysis D. Client with right empyema who has a chest tube and a fever of 103.2° F

What are possible causes of pulmonary empyema?

The most common cause of empyema is pulmonary infection, lung abscess, or infected pleural effusion. Pneumonia or lung abscess can spread across the pleura. Lymph node obstruction can cause a retrograde (backward) flood of infected lymph into the pleural space. A liver abscess or abdominal abscess can spread through the lymphatic system into the lungs. Chest surgery or trauma can introduce bacteria directly into the pleural space, leading to empyema. Blood from trauma may collect in the pleural space, promoting infection.

ANS: C The bacillus Calmette-Guérin (BCG) vaccine contains attenuated tubercle bacilli and is used in many countries to produce increased resistance to TB. The nurse will have a positive skin test. The client should be evaluated for TB with a chest x-ray. A physician examination is not necessary.

The newly employed nurse received a bacillus Calmette-Guérin (BCG) vaccine before moving to the United States. The nurse needs to receive a tuberculin (TB) test as part of the pre-employment physical. What does the nurse do? a. The nurse should not receive the tuberculin test. b. The nurse will need a two-step TB test. c. The nurse will need a chest x-ray instead. d. A physician should examine the nurse before the TB test is given.

High fever & elevated WBC count

The nurse is assessing an older adult who has been diagnosed with bacterial pharyngitis. Which assessment finding is typically associated with this medical diagnosis, but may not be present in the older adult pt?

ANS: D Management of bacterial pharyngitis involves the use of antibiotics and the same supportive care provided for viral pharyngitis. Stress the importance of completing the entire antibiotic prescription, even when symptoms improve or subside. Failure to take all prescribed antibiotics is often the cause of recurrent infections. Although it is important for overall health that the client know his or her HIV status, it is not the highest priority intervention in the treatment plan. Allergens do not cause bacterial infections. Nasal cultures would not be a high priority unless the client had "failed" treatment with more than one antibiotic and was compliant with treatment.

The nurse is caring for a client with recurrent bacterial pharyngitis. Which is the nurse's highest priority intervention? a. Assess for symptoms of human immune deficiency virus (HIV). b. Ask about exposure to allergens. c. Perform nasal cultures. d. Teach the client about antibiotic therapy.

ANS: D The NAAT is a new rapid test for the diagnosis of tuberculosis (TB). Results are available in less than 2 hours. A positive test is conclusive for TB, and the client should be placed in isolation per facility policy. A client with a negative QuantiFERON gold test would not have tuberculosis. Likewise, a client with a negative AFB would not have tuberculosis. The client with fever and weight loss could have tuberculosis, but diagnostic tests would be needed because these are nonspecific manifestations

The nurse is caring for several clients on a respiratory floor. The nurse should place the client with which condition in isolation? a. Fever and weight loss b. Negative QuantiFERON TB gold test c. Negative acid-fast bacillus (AFB) stain d. Positive nucleic acid amplification test (NAAT)

Decreased strength of respiratory muscles

The nurse is making home visits to an older adult recovering from a hip fracture & identifies the priority patient problem of risk for respiratory infection. Which condition represents a factor of normal aging that would contribute to this increased risk?

Complete antibiotics as prescribed, rest, drink fluids, & minimize contact with crowds

The nurse is providing discharge instructions about pneumonia to a patient & family. Which discharge information is the nurse sure to include?

Increased WBCs

The nurse is reviewing lab results for a patient who has pneumonia. Which lab value does the nurse expect to see for this paitent?

WBC 12 to 18

The nurse is reviewing the lab results for an older adult pt with pneumonia. Which lab value frequently seen in pts with pneumonia may not be seen in this patient?

"I will take three drugs: isoniazid, rifampin, & pyrazinamide, then ethambutol may be added later"

The nurse is teaching a pt about the combination drug therapy that's used in the treatment of TB. Which pt statement indicates the nurse's instruction was effective?

ANS: A If a client is "not reliable," the risk is that the client will not take medications as required, causing spread of an organism that may become more drug resistant. The other answers are not correct.

The nurse is worried that a client who is not entirely reliable is being discharged home on therapy for multidrug-resistant tuberculosis. What strategy is the best to use for this client? a. Directly observed therapy b. IV drug administration c. Remaining in the hospital d. Isolation

Urinary retention

The nurse notes that a patient has a disorder that contraindicates drug therapy that the physician has just prescribed for symptomatic relief of allergic rhinitis. Which disorder does the nurse report to the physician as a contraindication?

Provides a mask to the visitor

The nurse notices a visitor walking into the room of a client on airborne isolation with no protective gear. What does the nurse do? A. Ensures that the client is wearing a mask B. Tells the visitor that the client cannot receive visitors at this time C. Provides a particulate air respirator to the visitor D. Provides a mask to the visitor

Considerations For Older Adults

The older adult with pneumonia has weakness, fatigue, lethargy, confusion, and poor appetite. Fever and cough may be absent, but hypoxemia is often present. The most common manifestation of pneumonia in the older adult patient is acute confusion from hypoxia. The WBC count may not be elevated until the infection is severe. Waiting to treat the disease until more typical manifestations appear greatly increases the risk for sepsis and death.

Seasonal Influenza

The patient with influenza often has a severe headache, muscle aches, fever, chills, fatigue, and weakness. Adults are contagious from 24 hours before symptoms occur and up to 5 days after they begin. Patients who are immunosuppressed may be contagious for several weeks. Sore throat, cough, and watery nasal discharge generally follow the initial symptoms for a week or longer. Most patients feel fatigued for 1 to 2 weeks after the acute episode has resolved. -Vaccinations for the prevention of influenza are widely available. -An attenuated virus is a live virus that has been altered to reduce its ability to cause an infection. -Viral infections do not respond to traditional antibiotic therapy. Antiviral agents may be effective for prevention & treatment of some types of influenza. Amantadine (Symmetrel) & rimantadine (Flumadine) have been effective in the prevention & treatment of influenza A, although strains of resistant organisms are increasing. Ribavirin (Virazole) has been used for severe influenza B. Two drugs that may shorten the duration of influenza A & influenza B are zanamivir (Relenza) which is used as an oral inhalant, & oseltamivir (Tamiflu), which is an oral tablet.

The client comes to the emergency department with a sore throat. Examination reveals redness and swelling of the pharyngeal mucous membranes. Which diagnostic test does the nurse expect will be requested first? A. Chest x-ray B. Complete blood count C. Tuberculosis (TB) skin test D. Throat culture

Throat culture

Laryngitis

Treatment focused on relief and prevention—voice rest, steam inhalation, increased fluid intake, throat lozenges Reduce use of tobacco and alcohol Activities that strain larnyx: Singing Cheering Public speaking Heavy lifting Whispering

Nonsurgical treatment of sinusitis

Treatment includes the use of broad-spectrum antibiotics, analgesics for pain and fever, decongestants, steam humidification, hot and wet packs over the sinus area, and nasal saline irrigations. Teach the patient to increase fluid intake unless another medical problem requires fluid restriction. If this treatment plan is not successful, he or she may need to be evaluated with sinus x-rays and CT scans.

What are the most common bacteria causing tonsillitis

Trept, Staph, Haemophilus influenzae and Pneumococcus.

Viral Pharyngitis how long are you contagious and how long does it last.

Viral pharyngitis is contagious for 2 to 3 days. Symptoms usually subside within 3 to 10 days after onset, and the disease is usually self-limiting.

s&s of pneumonia in the older adult

WBC may not be elevated until the infection is severe; waiting to treat disease until more clear manifestations arise increases the risk for sepsis and death; weakness, fatigue, lethargy, confusion, poor appetite, hypoxemia--->>acute confusion

Teaching coughing, deep-breathing exercises, & use of incentive spirometry.

What nursing intervention prevents the complication of pneumonia for a surgical pt?

ANS: A, C, E Treatment of sinusitis includes the use of broad-spectrum antibiotics (e.g., amoxicillin), analgesics for pain and fever, decongestants, steam humidification, hot and wet packs over the sinus area, and nasal saline irrigations. As complementary therapy, echinacea is recommended for the symptom of rhinitis. Antifungal medications, fluid restrictions, and staying in a dry climate are not recommended.

What teaching is appropriate for a client with acute rhinitis and sinusitis? (Select all that apply.) a. Using hot packs over the sinuses b. Fluid restriction c. Saline irrigations d. Staying in a dry climate e. Taking echinacea f. Antifungal medications

6 months

What's the minimum time period for treatment of TB?

What happens to your voice when you have a tonsilar abcess with Pharyngitis

When a tonsillar abscess occurs with pharyngitis, the patient may have a "hot potato" voice—a thickened voice of poor quality.

ANS: B Most decongestants work by increasing blood vessel constriction. This action increases peripheral vascular resistance and blood pressure. The client who already has hypertension may develop dangerously high blood pressure when taking a decongestant. The client who has a latex allergy, is taking oral contraceptives, or has type 1 diabetes would not be likely to be affected by the decongestant in such a life-threatening manner as the client who is hypertensive.

Which client does the nurse caution to avoid taking over-the-counter decongestants for manifestations of a cold or flu? a. Young man with a latex allergy b. Middle-aged woman with hypertension c. Teenage woman who is taking oral contraceptives d. Older man who has had type 1 diabetes mellitus for 20 years

Pleuritic chest pain

Which complication of pneumonia creates pain that increases on inspiration because of inflammation of the parietal pleura?

History of tobacco use

Which condition causes a patient to have the greatest risk for community-acquired pneumonia?

ANS: B Maintenance of an SaO2 of at least 95% is a clear goal that indicates that the client has adequate oxygenation. Absence of cyanosis and the presence of confusion are assessment factors that contribute to evaluation of oxygen; however, they are not absolute measures. Likewise, walking three times a day does not directly address oxygenation.

Which is the highest priority goal to set for a client with pneumonia? a. Absence of cyanosis b. Maintenance of SaO2 of 95% c. Walking 20 feet three times daily d. Absence of confusion

ANS: C Although influenza can lead to pneumonia, and preventing influenza with a flu shot reduces the risk for a secondary pneumonia, bacterial pneumonia can be acquired without influenza as a precipitating event and can be life threatening. Getting both injections will not protect the client from respiratory problems, nor will it prevent the client from being infectious to other people.

Which is the nurse's best response to an older adult client who is hesitant to take the pneumococcal vaccination and influenza vaccine in the same year? a. "You need both injections. A risk factor for getting pneumonia is infection with influenza." b. "Take both injections. They will protect you against respiratory problems for this year." c. "The flu shot may protect you against influenza but not against bacteria that cause pneumonia." d. "You should get the pneumococcal vaccination so you won't infect other people."

Early recognition and quarantine

Which method is the best way to prevent outbreaks of pandemic influenza? A. Avoiding public gatherings at all times B. Early recognition and quarantine C. Vaccinating everyone with pneumonia vaccine D. Widespread distribution of antiviral drugs

d. More than 95 percent

Which of the following is a normal value for oxygen saturation? a. Less than 60 percent b. 61 to 85 percent c. 86 to 95 percent d. More than 95 percent

b. Wheezes

Which of the following terms is used to describe violin- like sounds heard on chest auscultation? a. Crackles b. Wheezes c. Friction rub d. Stridor

Client with pulmonary tuberculosis who is receiving multiple medications

Which of these clients should the charge nurse assign to the LPN/LVN working on the medical-surgical unit? A. Client with group A beta-hemolytic streptococcal pharyngitis who has stridor B. Client with pulmonary tuberculosis who is receiving multiple medications C. Client with sinusitis who has just arrived after having endoscopic sinus surgery D. Client with tonsillitis who has a thick-sounding voice and difficulty swallowing

Confused patient Pt with gram-negative colonization of the mouth Malnourished patient

Which patients are at risk for developing hospital-acquired pneumonia?

An alcoholic homeless man who occasionally stays in a shelter. A person with immune dysfunction or HIV Foreign immigrants (esp. those from the Philippines & Mexico)

Which people are at greatest risk for developing TB in the U.S?

ANS: B Although age is a factor in the development of community-acquired pneumonia, other lifestyle and exposure factors increase the risk to a greater extent than age. Two conditions that heavily predispose to the development of pneumonia are cigarette smoking and alcoholism. Dietary choices typically do not predispose to the development of pneumonia. Cigarette smoking interferes with the ciliary function of removal of invasive materials. Alcoholism usually results in unbalanced nutrition, as well as decreased immune function. A middle-aged adult, an older adult with wheezing induced by exercise, and a young adult vegetarian would not be at risk for community-acquired pneumonia because they have no predisposing conditions. pg 648

Which person is at greatest risk for developing a community-acquired pneumonia? a. Middle-aged teacher who typically eats a diet of Asian foods b. Older adult who smokes and has a substance abuse problem c. Older adult with exercise-induced wheezing d. Young adult aerobics instructor who is a vegetarian

32 year old trauma pt on a mechanical ventilator

Which pt is @ highest risk of developing pneumonia?

Negative airflow rooms are required for these patients Health care workers must wear a N95 or high-efficiency particulate air (HEPA) mask Gown & gloves are included in appropriate barrier protection

Which statement about the precautions of caring for a hospitalized patient with TB are true?

A serious inflammation of the bronchioles from various causes

Which statement best describes pneumonia?

Induration of 12 mm & positive sputum

Which test result indicates a patient has clinically active TB?

Can viruses cause tonsillitis

Yes

Isoniazid (INH), rifampin (Rifadin), pyrazinamide (Tebrazid), ethambutol (Myambutol)

Your client has been homeless and has spent the past 6 months living in shelters. The client has been diagnosed with confirmed tuberculosis (TB). You are completing your medication teaching with this client. About which medications will the nurse teach the client? A. Isoniazid (INH), rifampin (Rifadin), pyrazinamide (Tebrazid), ethambutol (Myambutol) B. Metronidazole (Flagyl), acyclovir (Zovirax), flunisolide (AeroBid), rifampin (Rifadin) C. Prednisone (Prednisone), guaifenesin (Organidin), ketorolac (Toradol), pyrazinamide (Trabrazide) D. Salmeterol (Serevent), cromolyn sodium (Intal), dexamethasone (Decadron), isoniazid (INH)

Focused assessment for a patient recovering from pneumonia

aChart 33-7 FOCUSED ASSESSMENT The Patient Recovering from Pneumonia Ask whether the patient has had any of these: •New-onset confusion •Chills •Fever •Persistent cough •Dyspnea •Wheezing •Hemoptysis •Increased sputum production •Chest discomfort •Increasing fatigue •Any other symptoms that have failed to resolve Assess the patient for: •Fever •Diaphoresis •Cyanosis, especially around the mouth or conjunctiva •Dyspnea, tachypnea, or tachycardia •Adventitious or abnormal breath sounds •Weakness

What patient education should the nurse provide to a TB patient regarding their medication?

adhere to the drug regimen- stress importance of taking each drug regularly as prescribed, for as long as prescribed-this ensures suppression of the disease.

How is TB transmitted

aerolization, active TB droplet

What is the transmission mode of tonsillitis

airborne

Peritonsillar Abscess PTA

also known as quinsy Complication of acute tonsillitis Manifestations: Pus causing one-sided swelling with deviation of the uvula Trismus ( tonic contraction of the muscles of mastication) difficulty breathing Bad breath, swollen lymph nodes

Rhinitis

an inflammation of the nasal mucosa -It can be caused by infection (viral or bacterial) or contact with allergens.

Dx of TB

anyone with a persistent cough or s/s compatible with TB weight loss, anorexia, NIGHT SWEATS, hemoptsyis, SOB, low grade fever (usually in the afternoons) or chills, irregular menses. purulent sputum possibly blood tinged Not infectious until starting to show symptoms

Non-environmental causes of pneumonia

bacteria viruses mycoplasmas fungi rickettsiae protozoa helminths (worms)

Cholinergic antagonists

bronchodilation by inhibiting the parasympathetic nervous system; Ipratropium (atrovent): 2-4 inh 4-6x/d (18mcg/dose)

Describe the initial infection of TB and where is it most often seen?

can last for years or decades before clinical symptoms develop. An infected person is not contagious to others until manifestations of disease occur. Seen in the middle or lower lobes of the lung

Beta-adrenergic agents

cause bronchodilation by relaxing smooth muscle; albuterol-1-2 inh q4-6h (90mcg/dose), salmeterol-2 inh q12h (25-50 mcg/dose)

s&s of pneumonia

chills, fever, persistent cough, dyspnea, wheezing, hemoptysis, chest discomfort, increasing fatigue, new onset confusion, diaphoresis, cyanosis, tachypnea, tachycardia, weakness, adventitious breath sounds

In the patient with COPD, an ABG reveals

chronic respiratory acidosis, compensatory alkalosis---->> kidneys retain bicarb

Treatment for Anthrax

ciprofloxacin (Cipro) or doxycycline (Vibramycin) or Amoxicillin (Amoxil or Trimox). Prophylaxis for 60 days or longer

People at greatest risk for TB

constant frequent contact with infected persons immune dysfunction or HIV crowed areas like: long term care facilities, prisons, MH facilities Older and homeless persons injection drugs or alcohol Foreign immigrants (Mexico, Phillipines and Vietnam)

Corticosteroids

disrupt all known pathways of inflammatory mediators, so the prevent inflammation; Fluticasone (flovent) 50 mcg MDI bid; Prednisone: 1-40 mg PO qd

How is SARS transmitted

droplet. people hacking up on you. it can contaminate surfaces but won't live long.

Physical examination of the chest is not provide conclusive evidence of TB, what else can help determine the presence of TB?

dullness with percussion, bronchial breath sounds & crackles; partial obstruction of bronchi or compression by lymph nodes can produce localized wheezing

COPD includes

emphysema and chronic bronchitis

What is considered to be front-line therapy for TB?

isoniazid and rifampin, pyrazinamide, and ethambutol

Emphysema

loss of lung elasticity and hyperinflation of the lung--->>dyspnea & increased RR, faster production of CO2 than can be expelled--->>chronic CO2 retention and respiratory acidosis

Hgb range

male: 14-18 g/dl female: 12-16 g/dl

RBC

male: 4.7-6.1 female: 4.2-5.4

Hct range

male: 42-52% female: 37-47%

TB is highly communicable and caused by...

mycobacterium tuberculosis

Caseastion necrosis

necrotic tissue being turned into a granular mass

typically, how much O2 should be administered to the patient with COPD?

no more than 2L/min; COPD patients drive to breath if from lack of O2, not from excess of CO2; too much O2 will take away their drive to breathe

Perennial rhinitis

occurs either intermittently with no seasonal pattern or continuously whenever the person is exposed to an offending allergen such as dust, animal dander, wool, or foods

what are the risk factors for health care-acquired pneumonia?

older adult, chronic lung dz, positive gram(-) colonization in the mouth, throat and stomach; aloc, recent aspiration, endotracheal, tracheostomy, or ng tube, poor nutrition, immunocompromised, meds that increase gastric pH, ventilation

Risk factors for contracting influenza or pneumonia (community-acquired)

older adult, never received vaccine or in last year, chronic health problem, uses tobacco or alcohol, recent exposure to respiratory or viral influenza

If a patient reports that he or she received the BCG vaccine within the last 10 years, what is most likely to occur?

patient will have a positive TB result; pt will need a chest xray to further determine the presence of active TB

When obtaining a patient's history, what should be addressed to assess a patient's exposure to TB?

patient's country of origin, traveling to other countries, previous TB results, if they have been in contact with someone with TB or previous BCG vaccine.

Ghon's tubercle

primary lesion, shown on xrays

What are the 2 most common pathogens that cause VAP

pseudomonas aeruginosa and acinetobacter

Most important areas to monitor avian flu during rehydration

pulse rate and quality of URINE OUTPUT

Describe secondary TB and where is it most often seen?

reactivation of the disease in a previously infected person. most likely occurs in older adults and those with HIV because their decreased immunity; upper lobes of the lung are most common site of reactivation, referred to as Simon's Foci

What happens to the red blood cells in pneumonia and how does that spread infection

red blood cells and fibrin move into the alveoli. The capillary leak (from white blood cell migration previously) spreads the infection to other areas of the lung. If gets into the bloodstream SEPSIS. Infection gets into the pleural cavity EMPYEMA results.

When should a patient with PTA go to the ER

s/s of obstruction drooling stridor

TB has a ____ onset

slow

Miliary/hematogenous TB

spread of TB throughout the body when a large number of organisms enter the blood; seen as tiny nodules scattered throughout the lung on chest xrays. The brain, liver, kidney, or bone marrow can become infected as a result of this spread.

Cromones

stabilizes the membranes of mast cells and prevent the release of inflammatory mediators. Nedocromil (Tilade) 4 mg MDI q6h

Early detection depends upon ________ rather than ______

subjective findings; observable symptoms/objective findings

Most important avian flu

supporting patient and prevent the spread of infection

Sintitis Surgery management nasal antral window

surgeon makes an opening in the front portion of the lower nasal bone to improve drainage through the nares

Residual Volume

the amount of air left in the lungs at maximum expiration

Total lung capacity

the amt of air contained within the lungs at maximum inspiration

Once therapy is intiated, sputum specimens are needed...

usually every 2-4 weeks

Ventilator-associated pneumonia (VAP)

ventilator associated pneumonia

What pulmonary function tests are monitored?

vital capacity, residual volume, total lung capacity

Intervention for laryngitis

voice rest, steam inhalations, increase fluid intake and throat lozenges. HCP antibiotics and bronchodialtors if also sinusitis, bronchitis or other bacterial infections are present

In addition to a persistent cough, a patient with TB may also exhibit:

weight loss, anorexia, calcification in the lungs, night sweats, hemoptysis, shortness of breath, fever, or chills

A 70-year-old client has a complicated medical history including chronic obstructive pulmonary disease (COPD). Which client statement indicates the need for further teaching about the disease? A. "I am here to receive the yearly pneumonia shot again." B. "I am here to get my yearly flu shot again." C. "I should avoid large gatherings during cold and flu season." D. "I should cough into my upper sleeve instead of my hand."

"I am here to receive the yearly pneumonia shot again."

Pharyngitis

"Sore throat" is common inflammation of pharyngeal mucous membranes Most sore throats in adults are viral Assess for: Odynophagia, dysphagia, fever, hyperemia Strep throat can lead to serious medical complications! Epiglottitis is rare complication Priority intervention: teaching about abx. Therapy When a tonsillar abscess occurs with pharyngitis, the patient may have a "hot potato" voice—a thickened voice of poor quality. Bacterial infections are more often associated with enlarged red tonsils, exudate, purulent nasal discharge, and local lymph node enlargement

Acute viral rhinitis

(coryza, or the common cold) is caused by any of over 200 viruses. It spreads from person to person by droplets from sneezing or coughing and by direct contact. Colds are most contagious in the first 2 to 3 days after symptoms appear.

Inhalation anthrax

(respiratory anthrax) is a bacterial infection caused by the gram-positive organism Bacillus anthracis, which lives as a spore in contaminated soil. Infection with this organism occurs through the skin, the intestinal tract, or the lungs. Inhalation anthrax is a rare natural occurrence in the United States and is nearly 100% fatal without treatment. It is not spread by person-to-person contact.

Health Promotion & Maintenance

-23 different types of pneumonia organisms are included in the pneumococcal polysaccharide vaccine (PPV23). -Patient education is important in the prevention of pneumonia. Encourage people older than 65 years & those with a chronic health problem to receive the PPV23. -Strict handwashing to avoid the spread of organisms & avoiding large gathering of people during cold & flu season.

Risk Factors for Pneumonia

-Community-Acquired Pneumonia *older adult *never received the pneumococcal vaccination or received it more than 6 years ago *Didn't receive the influenza vaccine in the previous year *Has a chronic health problem or other coexisting condition *Has recently been exposed to respiratory viral or influenza infections *Uses tobacco or alcohol or is exposed to high amounts of secondhand smoke -Health Care-Acquired Pneumonia *An older adult *Has chronic lung disease *Has presence of gram-negative colonization of the mouth, throat, & stomach *Has an altered level of consciousness *Has had a recent aspiration event *Has presence of endotracheal, tracheostomy, or nasogastric tube *Has poor nutritional status *Has immunocompromised status (from disease or drug therapy) *Uses drugs that increase gastric pH (histamine blockers, antacids) or alkaline tube feedings. *Currently receiving mechanical ventilation (ventilator-associated pneumonia)

Etiology of Pneumonia

-Develops when the immune system can't combat the virulence of the invading organisms. -Can be caused by bacteria, viruses, mycoplasmas, fungi, rickettsiae, protozoa, & helminths (worms). -Noninfectious causes of pneumonia include: inhalation of toxic gases, chemical fumes, & smoke & aspiration of water, food, fluid, & vomitus.

Pandemic Influenza

-Many viral infections among animals & birds are not usually transmitted to humans. -Animal & bird viruses mutated & became highly infectious to humans. These infections are termed pandemic because they have the potential to spread globally. -H1N1 influenza. -The antiviral drugs oseltamivir (Tamiflu) & zanamivir (Relenza) should be widely distributed. These drugs are not likely to prevent the disease but may reduce the severity of the infection & reduce the mortality rate. -Strict isolation. -Care of the patient with avian influenza focuses on supporting the patient & preventing spread of the disease. Both are equally important. -Manifestations of avian influenza are similar to other respiratory infections- cough, fever, & sore throat. These progress rapidly to shortness of breath & pneumonia. In addition, diarrhea, vomiting, abdominal pain, & bleeding from the nose & gums occur. -Ask any patient with these symptoms if he or she has recently (within the past 10 days) traveled to area of the world affected by H5N1. If such travel has occurred, coordinate with the health care team to place the patient in an airborne isolation room with negative air pressure. The precautions remain until the diagnosis of H5N1 is ruled out or the threat of contagion is over. -Fit-tested respirator or a standard surgical mask. -Use the antiviral drug oseltamivir (Tamiflu) or zanamivir (Relenza) within 48 hours of contact with the infected patient. -No effective treatment for this infection currently exists. -Antibiotics are used to treat a bacterial pneumonia that may occur with H5N1. -H5N1- patient may have severe diarrhea & need fluid therapy. -The 2 most important areas to monitor during rehydration are pulse rate & quality & urine output.

Diagnostic Assessments

-New Test* the fully automated nucleic acid amplification test (NAAT) for TB. Results are available in less than 2 hours. -Diagnosis is suggested by the manifestations & a positive smear for acid-fast bacillus. Sputum is obtained, smeared on a slide, & stained with a red dye. After the slide has dried, it's treated with an acid alcohol to remove the stain. TB does not de-stain with this procedure & remains red. -Blood analysis by an enzyme-linked immunosorbent assay using the QuantiFERON-TB Gold (QFT-G) is a relatively rapid test for the presence of M. tuberculosis. -Sputum culture confirms the diagnosis. Enhanced TB cultures & automated mycobacterial cultures require 1 to 4 weeks to determine a positive or negative result. -The tuberculin test (Mantoux test)- most commonly used reliable test of TB infection. A small amount (0.1 mL) of purified protein derivative (PPD) is placed intradermally in the forearm. An area of induration (localized swelling with hardness of soft tissue) not just redness, measuring 10 mm or greater in diameter 48 to 72 hours after injection indicates exposure to & possible infection with TB. -A positive reaction does not mean that active disease is present but indicates exposure to TB or the presence of inactive (dormant) disease. -A reduced skin reaction or a negative skin test does not rule out TB disease or infection of the very old or anyone who is severely immunocompromised. Failure to have a skin response because of reduced immune function when infection is present is called anergy.

Inhalation Anthrax

-Prodromal Stage (early) Fever, fatigue, mild chest pain, dry cough, no manifestations of upper respiratory infection, mediastinal "widening" on chest xray -Fulminant Stage (late) Sudden onset of breathlessness, dyspnea, diaphoresis, stridor on inhalation & exhalation; hypoxia; high fever; mediastinitis; pleural effusion; hypotension; septic shock

Interventions

-Rest, increase fluid intake; humidify the air; & use analgesics for pain. -Management of bacterial- involves antibiotics. -Streptococcal infection- oral penicillin or cephalosporin- Drugs from macrolide class (azithromycin or erythromycin) used if patient is allergic to penicillin. -If bacterial does not improve with antibiotics- should consider HIV testing. -Rare complication of pharyngitis in adults if infection of the epiglottis & supraglottic structures (epiglottitis). The epiglottis is a flaplike structure that closes over the trachea during swallowing to prevent aspiration. An inflamed epiglottis can swell & obstruct the airway, causing an emergency that inhibits oxygenation & tissue perfusion. **CRITICAL RESCUE** -If a patient with pharyngitis develops stridor or other indications of airway obstruction; notify the Rapid Response Team. *Difficulty breathing*stridor*or drooling occurs*

Complications of group A Streptococcal Infection

-Rheumatic fever -Acute glomerulonephritis -Peritonsillar abscess -Retropharyngeal abscess -Otitis media -Sinusitis -Mastoiditis -Bronchitis -Pneumonia -Scarlet fever

Laboratory & Imaging & other Diagnostic Assessments

-Sputum is obtained & examined by Gram stain, culture, & sensitivity testing. -The responsible organism often is not identified. -A CBC is obtained to assess an elevated WBC count, which is a common finding except in older adults. Blood cultures may be performed to determine whether the organism has invaded the blood. -In severely ill patients, arterial blood gases (ABGs) may be performed to determine baseline arterial oxygen & carbon dioxide levels & help identify a need for supplemental oxygen. Serum electrolyte, blood urea nitrogen (BUN) & creatinine levels also are assessed. A high BUN level may occur as a result of dehydration. Hypernatremia (high blood sodium levels) occurs with dehydration as a result of fever & decreased fluid intake! Chest x-ray continues to be the most common diagnostic test for pneumonia but may not show changes until 2 or more days after manifestations are present. It usually appears on chest x-ray as an area of increased density. It may involve a lung segment, a lobe, one lung, or both lungs. In the older adult, the chest x-ray is essential for early diagnosis because pneumonia symptoms are often vague. -Pulse ox is used to assess for hypoxemia. -Invasive tests such as transtracheal aspiration, bronchoscopy, or direct needle aspiration of the lung may be needed. -Thoracentesis is most often used in patients who have an accompanying pleural effusion.

Assessment of TB

-TB has a slow onset & patients are not aware of symptoms until the disease is advanced. *TB should be considered for any patient with a persistent cough or other symptoms compatible with TB, such as weight loss, anorexia, night sweats, hemoptysis, shortness of breath, fever or chills. Ask if patient has had Calmette-Guerin (BCG) vaccine. -The BCG vaccine contains attenuated tubercle bacilli and is used in many countries to produce increased resistance to TB. Anyone who has received BCG vaccine within the previous 10 years will have a positive skin test that can complicate interpretation. Usually the size of the skin response decreases each year after BCG vaccination. These patients should be evaluated for TB with a chest x-ray or the QuantiFERON-TB Gold test. The effectiveness of BCG vaccine in preventing TB is controversial, and it is not used widely for this purpose in the United States or Canada. -Patient with TB has progressive fatigue, lethargy, nausea, anorexia, weight loss, irregular menses, & a low-grade fever. Manifestations may have been present for weeks or months. Night sweats may occur with the fever. Patient has a cough & mucopurulent sputum, which may be streaked with blood. Chest tightness & a dull, aching chest pain occur with the cough. -Dullness with percussion may be heard over the involved lung fields, as may bronchial breath sounds, crackles, & increased transmission of spoken or whispered sounds. Partial obstruction of a bronchus from endobronchial disease or compression by lymph nodes may produce localized wheezing.

Ventilator Associated Pneumonia

-on the rise, esp. among patients with endotracheal tubes in place for mechanical ventilation. -Three care actions- "ventilator bundle"- have been shown to reduce the incidence of VAP- hand hygiene, oral care, & head-of-bed elevation. -Oral care is critical in reducing the risk because many common organisms causing VAP are translocated from the patient's mouth into the respiratory tract. *If possible perform oral care with a disinfecting oral rinse right before the intubation. *Don't wear hand jewelry, esp. rings, when providing care to ventilator patients. *Wash hands before & after contact with the patient. *Provide complete oral care at least every 12 hours. *Remove subglottic secretions frequently (@ least every 2 hours) or continuously (when the endotracheal tube has a separate lumen that opens directly about the tube cuff) *Keep the HOB elevated to at least 30 degrees unless another health problem is a contraindication for this position.

How does the process of TB infections occur?

1. The granulomatous inflammation created by the tubercle bacillus in the lung becomes surrounded by collagen, fibroblasts, and lymphocytes. 2. Caseation necrosis, which is necrotic tissue being turned into a granular mass, occurs in the center of the lesion. If this area shows on x-ray, it is called Ghon's tubercle, or the primary lesion. -Areas of caseation then undergo resorption, degeneration, & fibrosis. These necrotic areas may calcify (calcification) or liquefy (liquefaction). If liquefaction occurs, the liquid material then empties into a bronchus & the evacuated area becomes a cavity (cavitation). Bacilli continue to grow in the necrotic cavity wall & spread via lymph channels into new areas of the lung.

When taking an IV prophylaxis for anthrax, treatment lasts for...

7 days, when patient response is good and they show improvement, they are changed over to oral treatment that lasts 60 days

Need to complete penicillin or penicillin-like antibiotics

A child's diagnosed with a group B Streptococcus throat infection. In teaching the parents about treatment of the infection, what does the nurse instruct the parents?

ANS: D Untreated or ineffectively treated peritonsillar abscesses can extend throughout the pharyngeal area, causing swelling that may jeopardize the client's airway. Therefore, the client should take his antibiotic for the entire time prescribed to maximize the therapeutic effect. Gargling with warm water and refraining from normal activities may provide symptomatic relief for the client but would not be considered priority instructions. Also, swelling, pain, and inflammation could be noted by the client on the same side of the neck as the abscess. pg 644

A client has a peritonsillar abscess. Which priority instruction does the nurse provide to this client? a. "If you notice an enlarged node on the side of your neck where the abscess is, call your health care provider." b. "Stay home from work or school until your temperature has been normal for 24 hours." c. "You may gargle with warm water that has a teaspoon of salt in it as often as you like." d. "Take the antibiotic for the entire time it is prescribed, not just until you feel better."

ANS: A Bacterial infection often occurs with acute rhinitis. The nurse should assess for symptoms because treatment may be warranted. It is not essential to assess for allergies or the use of nasal spray, or to determine whether drug use is occurring. All of these interventions are focused on determining a cause for repeated acute rhinitis and are primarily the responsibility of the health care provider. The nurse should focus on client assessment and should determine whether a secondary infection is present.

A client has acute rhinitis. What is the most important intervention for the nurse to perform? a. Assess for symptoms of infection. b. Ascertain whether the client has allergies. c. Question the client on the use of nasal sprays. d. Do blood and urine screenings for drug use.

ANS: C The client should wear a mask when out of the home environment and in crowds to prevent spread of the infection. The other statements are not accurate.

A client has multidrug-resistant tuberculosis (TB). What is the most important fact for the nurse to teach the client? a. "You will need to take medications longer than clients with other strains." b. "You will need to remain in the hospital until cultures are negative." c. "You will need to wear a mask when you go out in public." d. "You will need to have drug cultures done weekly."

ANS: D Generally a rash can appear with bacterial pharyngitis, but not with viral. The other symptoms are characteristic of both. ph 643

A client has pharyngitis. Which symptom helps the nurse determine whether the infection is bacterial versus viral? a. Redness in the back of the throat b. Enlarged lymph glands in the neck c. Nasal discharge d. Skin rash

Place the client in a negative air pressure room.

A client is admitted to the emergency department (ED) with a possible diagnosis of avian influenza ("bird flu"). Which of these actions included in the hospital protocol for avian influenza will the nurse take first? A. Ensure that ED staff members receive oseltamivir (Tamiflu). B. Obtain specimens for the H5 polymerase chain reaction test. C. Place the client in a negative air pressure room. D. Start an IV line and administer rehydration therapy.

ANS: B The client with pneumonia may have dullness to percussion on the affected side. The other options are all inconsistent with pneumonia.

A client is admitted with left lower lung pneumonia. Which assessment finding does the nurse correlate with this condition? a. Expiratory wheeze on the right side b. Dullness to percussion on the lower left side c. Crepitus of the skin around the left lung d. Crackles heard on expiration bilaterally

ANS: B The client who is experiencing avian influenza should be on both airborne and contact isolation. Standard antibiotic agents would be ineffective with this disease process, as would most of the standard antiviral medications commonly used for influenza. Human-to-human contact through family members is likely only in very close living arrangements, so only specific members of the client's family should consider diagnostic testing.

A client is admitted with suspected avian influenza. The family asks the nurse what kind of care the client will get. Which statement by the nurse is correct? a. "He will be given standard antibiotic agents and will be placed in contact isolation." b. "He will be placed on airborne and contact isolation." c. "Oseltamivir (Tamiflu) will reduce complications of this infection." d. "All family members should be tested for evidence of the same disease."

The client may be switched from IV to oral antibiotics in 2 to 3 days.

A client is being admitted for pneumonia. The sputum culture is positive for Streptococcus. The client asks about the length of the treatment. On what does the nurse base the answer? A. The client may be switched from IV to oral antibiotics in 2 to 3 days. B. Usually anti-infectives are used for 7 to 10 days. C. When the client has completed 6 days of therapy D. When the client is afebrile for 24 hours

ANS: B Vaccines for influenza are widely available and are recommended to prevent flu. Flu continues to be a major problem, affecting up to 20% of the U.S. population and causing 36,000 deaths annually. pg 645

A client is worried about contracting influenza. What is the nurse's best response to the client? a. "Flu is no longer a prevalent problem." b. "Did you receive a flu vaccine this year?" c. "Current flu strains are generally mild." d. "If you develop symptoms, antibiotics will cure you."

ANS: A, C, F Taking the daily dose of medications at bedtime may help to decrease nausea. A well-balanced diet with foods rich in iron, protein, and vitamins C and B also helps to decrease nausea. Antiemetics are often prescribed. Drinking fluids with medications should not influence the nausea; neither should taking medications with milk. Spacing medications 12 hours apart is not recommended therapy.

A client started on therapy for tuberculosis infection is reporting nausea. What does the nurse teach this client? (Select all that apply.) a. Eat a diet rich in protein, iron, and vitamins. b. Do not drink fluids with medications. c. Take medications at bedtime. d. Space medications 12 hours apart. e. Take medications with milk. f. Take an antiemetic daily.

ANS: A The nurse should first assess the throat for signs of peritonsillar abscess. If present, the nurse should call the health care provider immediately because aspiration of the abscess may be needed to maintain the airway.

A client who has had acute tonsillitis develops drooling and reports severe throat pain. What is the nurse's priority intervention? a. Assess the throat for deviation of the uvula. b. Prepare the client for surgery. c. Teach the client about antibiotic therapy. d. Prepare the client for percutaneous needle aspiration.

ANS: C Immune compromised clients are contagious for several weeks. The client should remain at home until he is not contagious. pg 645

A client who is immune compromised develops muscle aches and fever. The client is admitted to the hospital for several days and is diagnosed with influenza. At discharge, the client asks when he can go back to work. What is the nurse's best response? a. "You should be able to return to work in 5 days." b. "You can return to work as soon as you feel ready." c. "You cannot return to work for several weeks." d. "You will need to have cultures performed before returning to work."

ANS: A, B, E TB symptoms include nausea and weight loss, as well as night sweats. Inability to sleep and ankle edema are not typical symptoms. Increased urination also is not a typical symptom. pg 654-655

A client who previously had a bacillus Calmette-Guérin (BCG) vaccine has a positive tuberculosis (TB) test. What symptoms assist in determining that the client has active disease? (Select all that apply.) a. Nausea b. Weight loss c. Insomnia d. Ankle edema e. Night sweats f. Increased urination

ANS: C The client who works in a day care facility and is infected with Streptococcus pneumoniae may have a drug-resistant pneumonia. It is extremely important that this organism does not spread to other clients; the client should be isolated.

A client who works in a day care facility is admitted to the emergency department. The client is diagnosed with pneumonia, and a sputum culture is taken. Infection with Streptococcus pneumoniae is confirmed. What is the nurse's primary action? a. Have emergency intubation equipment nearby. b. Teach the client about the treatment. c. Isolate the client. d. Perform chest physiotherapy.

Draw aerobic and anaerobic blood cultures

A client with pneumonia caused by aspiration after alcohol intoxication has just been admitted. The client is febrile and agitated. Which physician order should the nurse implement first? A. Administer levofloxacin (Levaquin) 500 mg IV. B. Draw aerobic and anaerobic blood cultures. C. Give lorazepam (Ativan) as needed for agitation. D. Refer to social worker for alcohol counseling.

Surgical treatment of sinusitis

A common procedure used for sinusitis that does not respond to drug therapy is functional endoscopic sinus surgery (FESS). Small endoscopes (sinoscopes) are used to first visualize the area. Instruments attached to the sinoscopes are used to open the nasal ostium and remove infected mucosa or improve the pathway for nasal drainage. A balloon catheter may also be used during FESS to change the shape of the nasal ostium to relieve obstruction and promote drainage. Mucosal healing- 4 to 6 weeks.

Peritonsillar abscess (PTA)

A complication of acute tonsillitis. The infection spreads from the tonsil to the surrounding tissue, which forms an abscess. The most common cause of PTA is group A beta-hemolytic Streptococcus. -Manifestations: pus behind the tonsil causing one-sided swelling with deviation of the uvula toward the unaffected side. Patient may drool, have severe throat pain radiating to the ear, have a voice change, & have difficulty swallowing. Bad breath is present, & lymph nodes on the affected side are swollen. An intraoral or transcutaneous ultrasound may be used for diagnosis. -Outpatient management with antibiotic therapy & percutaneous needle aspiration of the abscess is usually needed. -Acute management: IV opioid analgesics for severe pain & IV steroids to reduce swelling. -Hospitalization is required when the airway is in jeopardy or when the infection does not respond to antibiotic therapy. Incision and drainage of the abscess and additional antibiotic therapy may be needed. A tonsillectomy may be performed to prevent recurrence.

Pao2 is 90 mm Hg with crackles

A critical concern for a pt returning to the unit after a surgical procedure is related to impaired oxygenation caused by inadequate ventilation. Which ABG value & assessment finding indicates to the nurse that oxygen & incentive spirometry must be administered?

Tuberculosis (TB)

A highly communicable disease caused by Mycobacterium tuberculosis. It is the most common bacterial infection worldwide. The organism is transmitted via aerosolization (i.e., an airborne route) Far more people are infected with the bacillus than actually develop active TB.

Ciprofloxacin (Cipro) 400 mg IV every 12 hours

A local hunter is admitted to the intensive care unit with a diagnosis of inhalational anthrax. Which of the following medications will the RN anticipate as a physician request? A. Amoxicillin (Amoxil, Triamox) 500 mg orally every 8 hours B. Ceftriaxone (Rocephin) 2 g IV every 8 hours C. Ciprofloxacin (Cipro) 400 mg IV every 12 hours D. Pyrazinamide (PZA) 1000 to 2000 mg orally every day

Lung Abscess

A lung abscess is a localized area of destruction caused by liquefaction necrosis, which is usually related to pyogenic bacteria. Patients with an abscess often have a history of pneumonia, aspiration of stomach contents, or obstruction as a result of a tumor or foreign body. Multiple abscesses and cavities form in patients with tuberculosis (TB) or fungal infections of the lung.

Use reflection of light through the tissues, & observe for a red glow on the cheek

A patient comes to the walk-in clinic reporting seasonal nasal congestion, sneezing, rhinorrhea, & itchy watery eyes. The nurse identifies that the patient most likely has rhinitis & should also be assessed for sinusitis. How does the nurse assess for sinusitis?

Streptococcus

A patient reporting a "sore throat" also has a temp of 101.4 F, scarlatiniform rash, & a positive rapid test throat culture. This pt will most likely be treated for which type of bacterial infection?

Sinusitis

A patient reports difficulty breathing, facial pain (especially when head is dependent)., sneezing or coughing, green or bloody nasal drainage, productive cough, & low-grade fever. Which disorder does the nurse suspect?

Tuberculosis

A patient with HIV is admitted to the hospital with a temperature of 99.6 F & reports of bloody sputum, night sweats, feeling of tiredness, & SOB. What are these assessment findings consistent with?

Isoniazid (INH)

A pt has an HIV infection, but the TB skin test shows an induration of less than 10 mm and no clinical symptoms of TB are present. Which medication does the pt receive for a period of 12 months to prevent TB?

3 negative sputum cultures

A pt has been compliant with drug therapy for TB & has returned as instructed for follow up. Which result indicates that the pt is no longer infectious/communicable?

Administer respiratory therapy in a timely manner to decrease bronchospasms.

A pt hospitalized for pneumonia has the priority pt problem of Ineffective Airway Clearance related to fatigue, chest pain, excessive secretions, & muscle weakness. What nursing intervention helps to correct these problems?

Cough, clear sputum, temp 99F, pulse ox @ 96% on room air

A pt is admitted to the hospital for treatment of pneumonia. Which nursing assessment finding best indicates that the pt is responding to antibiotics?

HEPA respirator mask

A pt is admitted to the hospital to rule out TB. What type of mask does the nurse wear when caring for this patient?

Standard precautions & no respiratory isolation

A pt is admitted to the hospital to rule out pneumonia. Which infection control technique does the nurse maintain?

Patchy areas of consolidation

A pt is admitted to the hospital with pneumonia. What does the nurse expect the chest x-ray to reveal?

Start broad-spectrum IV antibiotic therapy without delay.

A pt is admitted to the hospital with pneumonia. Which approach to the administration of antibiotics does the nurse expect the physician to order?

Dyspnea Hypoxemia Chest discomfort

A pt is seen in the HCP's office & is diagnosed with community-acquired pneumonia. What are the most common symptoms the pt will have?

1 week

A pt reporting a soreness in the throat is diagnosed with "strep throat". To prevent complications such as rheumatic heart disease, this pt should receive antibiotic treatment within what time frame?

-Clean all respiratory equipment you have @ home. -Avoid indoor pollutants such as dust & aerosols. -Get plenty of rest & sleep daily.

A pt with COPD needs instructions in the measures to prevent pneumonia. What information does the nurse include?

Strict respiratory isolation & use of specially designed facemasks

A pt with suspected TB is admitted to the hospital. Along with a private room, which nursing intervention is appropriate related to isolation procedures?

Pleuritic chest pain

A stabbing pain upon taking a deep breath

What are the s/s of laryngitis

Acute hoarseness, dry cough, difficulty swallowing, temporary voice loss (APHONIA)

-Elevate HOB to between 30 and 45 degrees whenever possible. -Continuously remove subglottic secretions -Perform handwashing before & after contact with each patient -No rings are worn when caring for ventilator patients -Perform meticulous oral care no less frequently than every 12 hours

After a motor vehicle accident, the pt is dependent on mechanical ventilation. The nurse initiates "ventilator bundle" precautions to prevent ventilator-associated pneumonia. Which actions does the nurse perform to initiate these precautions?

Induration/hardened area measures 10 mm or greater

After receiving the subQ Mantoux skin test, a pt with no risk factors returns to the clinic in the required 48 to 72 hours for the test results. Which assessment finding indicates a positive result?

Fatigue Night sweats Low-grade fever

After several weeks of "not feeling well" a pt is seen in the physician's office for possible TB. If TB is present, which assessment findings does the nurse expect to observe?

Drugs for pneumonia

Amikacin (Amikin) Ampicillin/sulbactam (Unasyn) Azithromycin (Zithromax, Zmax) Aztreonam (Azactam) Cefuroxime (Ceftin, Kefurox, Zinacef) Erythromycin (E-Mycin, E.E.S., many others) Cefepime (Maxipime) Ceftriaxone (Rocephin) Cefotaxime (Claforan) Ceftazidime (Ceptaz, Fortaz, Tazicef, Tazidime) Ciprofloxacin (Ciloxan, Cipro) Clindamycin (Cleocin Phosphate, Cleocin) Clarithromycin (Biaxin) Dirithromycin (Dynabac) Doxycycline (Adoxa, many others) Gentamicin (Garamycin) Imipenem/Cilastatin (Primaxin) Levofloxacin (Iquix, Levaquin, Quixin) Linezolid (Zyvox) Meropenem (Merrem) Metronidazole (Flagyl, many others) Moxifloxacin (Avelox, Vigamox) Piperacillin/tazobactam (Zosyn) Teicoplanin (Targocid)* Ticarcillin/clavulanate (Timentin) Vancomycin (Vancocin)

Severe acute respiratory syndrome (SARS)

An easily spread respiratory infection first identified in China in November 2002. At first appearing as an atypical pneumonia, it is caused by a new, more virulent form of coronavirus, and there is no known effective treatment. These viruses have ribonucleic acid (RNA) as their genetic material & have many projections that look like a halo or "corona". This family of viruses causes many forms of the common cold. It infects cells of the respiratory tract, triggering inflammatory responses, and stays in the respiratory passageways rather than spreading into the blood. The virus is easily spread by airborne droplets from infected people through sneezing, coughing, and talking. People at greatest risk for SARS are those in close direct contact with an infected person. The portals of entry are the mucous membranes of the eyes, nose, and mouth.

Pneumonia

An excess of fluid in the lungs resulting from an inflammatory process. -The inflammation is triggered by many infectious organisms & by inhalation of irritating agents. -Infectious pneumonias are categorized as community-acquired pneumonia (CAP) or health care-associated pneumonia (known as HAP or HAI)- depending on where the patient was exposed to the infectious agent. -The inflammation occurs in the interstitial spaces, the alveoli, and often the bronchioles. The process begins when organisms penetrate the airway mucosa and multiply in the alveoli. White blood cells (WBCs) migrate to the area of infection, causing local capillary leak, edema, and exudate. These fluids collect in and around the alveoli, and the alveolar walls thicken. Both events seriously reduce gas exchange and lead to hypoxemia, interfering with oxygenation and possibly leading to death. Red blood cells (RBCs) and fibrin also move into the alveoli. The capillary leak spreads the infection to other areas of the lung. If the organisms move into the bloodstream, sepsis results; if the infection extends into the pleural cavity, empyema (a collection of pus in the pleural cavity) results. The fibrin and edema of inflammation stiffen the lung, reducing compliance and decreasing the vital capacity. Alveolar collapse (atelectasis) further reduces the ability of the lung to oxygenate the blood moving through it. As a result, arterial oxygen levels fall, causing hypoxemia -May occur as lobar pneumonia with consolidation (solidification, lack of air spaces) in a segment or an entire lobe of the lung or as bronchopneumonia with diffusely scattered patches around the bronchi!

Tonsillitis

An inflammation and infection of the tonsils and lymphatic tissues located on each side of the throat -A contagious airborne infection. -Diagnostic Tests: -Rapid Antigen Test (RAT), CBC, throat culture & sensitivity (C&S) & Monospot test. -The WBC count usually is elevated in bacterial infections & normal in viral infections. -Antibiotics (usually penicillin or azithromycin) are prescribed for 7 to 10 days. -Surgical intervention for tonsillitis may be needed for recurrent acute infections (especially group A beta-hemolytic streptococcal infections), chronic infections that do not respond to antibiotics, a peritonsillar abscess, and enlarged tonsils or adenoids that obstruct the airway. -The procedure may involve complete tonsil removal (for chronic infection) or a partial tonsil removal for obstruction without infection. The adenoids may be removed at the same time. -After surgery, nursing interventions focus on assessing for airway clearance, providing pain relief, & monitoring for excessive bleeding. **ACUTE TONSILLITIS** -Sudden onset of a mild to severe sore throat -Fever -Muscle aches -Chills -Dysphagia, odynophagia (painful swallowing of food) -Pain in the ears -Headache -Anorexia -Malaise -"Hot potato" voice (thickened voice of poor quality) -Tonsils visually swollen & red with pus -Tonsils may be covered with a white or yellow exudate -Purulent drainage may be expressed by pressing a tonsil -Uvula visually edematous or inflamed -Cervical lymph nodes usually tender & enlarged

Laryngitis

An inflammation of the mucous membranes lining the larynx and may or may not include edema of the vocal cords. -Can occur has a single problem or occur with upper respiratory infections. -Also can be a manifestation of other problems- such as throat or lung cancer. -Common causes include exposure to irritating inhalants (e.g., chemical fumes, tobacco, alcohol, smoke), voice overuse, inhalation of fumes (e.g., glue, paint thinner, butane), or intubation. Recurrent laryngitis may be caused by gastroesophageal reflux disease (GERD). -Assess patient for hoarseness, dry cough, & difficulty swallowing. Complete but temporary voice loss (aphonia) also may occur. -If suspicious lesions are present, an x-ray, computed tomography, or fiberoptic laryngoscopic examination may be needed. -Nursing management focuses: symptom relief & prevention. -Treatment- consists of voice rest, steam inhalations, increased fluid intake & throat lozenges. Antibiotic therapy & bronchodilators are prescribed when sinusitis, bronchitis, or other bacterial infection is also present. -Teach the patient and family about relief measures, infection prevention, and avoidance of alcohol, tobacco, and pollutants, which can irritate the larynx. Teach about preventive strategies such as reducing tobacco and alcohol use. Emphasize the need to avoid activities that place an added strain on the larynx, such as singing, cheering, public speaking, heavy lifting, and whispering. Speech-language therapy is used when vocal cord injury occurs with laryngitis. Further evaluation is needed for recurrent bouts of laryngitis.

ANS: A People older than 50 years and those with chronic disease should be vaccinated against the flu each year early in the fall because they are at higher risk of developing complications if they do get ill. Flu shots appear to be effective for only one flu season, so the client should get one annually. The live vaccine is recommended only for healthy people up to age 49. This vaccination should not have interactions with heart medications. pg 645

An older adult client with heart failure asks if she should get a flu shot. Which is the nurse's best response? a. "Yes, because of your heart failure you are at greater risk for complications." b. "Yes, if it has been longer than 5 years since your last flu vaccination." c. "No, your heart failure makes you too weak to get the live virus vaccine." d. "No, the vaccine will interact with your heart medications."

ANS: A Confusion in an older adult can signify hypoxia. If the nurse waited to intervene until the older adult showed more traditional symptoms of pneumonia, the client may become critically ill. The other manifestations also require intervention but not as the priority.

An older adult is admitted to the emergency department with respiratory symptoms. Which client symptom requires the nurse to intervene immediately? a. Confusion b. Scattered wheezing c. Crackles d. Flushed cheeks

A bronchodilator would help decrease the bronchospasm.

An older client presents to the emergency department (ED) with a 2-day history of cough, pain on inspiration, shortness of breath, and dyspnea. The client never had a pneumococcal vaccine. The client's chest x-ray shows density in both bases. The client has wheezing upon auscultation of both lungs. Would a bronchodilator be beneficial for this client? A. A bronchodilator would not be beneficial for this client. B. A bronchodilator would help decrease the bronchospasm. C. It would clear up the density in the bases of the client's lungs. D. It would decrease the client's pain on inspiration.

ANS: D The nurse needs to assess more before intervening. Clients often take antihistamines for a "cold." Antihistamines are often composed of anticholinergic drugs. In older adult clients, these medications can cause or worsen urinary retention.

An older client reports having a cold and a "full bladder." What does the nurse obtain for or from the client? a. Order for a Foley catheter b. Order for a one-time catheterization c. Urine specimen d. History focusing on current medications

Treatment for pandemic flu

Antiviral drugs oseltamivir (Tamiflu), zanamivir (Relenza) Not likely to prevent, but may reduce severity. Use within 48 hours of contact

Sinitis Surgical Management with a needle

Antral irrigation kown as maxillary antral puncture and lavage. Under local a large-gauge needle is inserted into the maxillary sinus on the affected side. Fluid or pus drained then irrigate with ns and antibiotic solution or both.

Pyrazinamide

Avoid drinking alcoholic beverages If going out into the sun, wear protective clothing & sunscreen

Inhalation Anthrax

Bacterial infection caused by Bacillus anthracis from contaminated soil Fatality rate 100% if untreated Two stages: prodromal stage, fulminant stage Drug therapy: ciprofloxacin, doxycycline, amoxicillin (Chart 33-9 in 7th ed.)

Bacterial Pharyngitis is more often associated with what

Bacterial infections are more often associated with enlarged red tonsils, exudate, purulent nasal discharge, and local lymph node enlargement

What labs should be assessed in the presence of pneumonia?

CBC, sputum, blood cultures, ABG, electrolytes, BUN and creatinine

Laryngitis causes

Can occur with a lower resp infection Exposure to irritating inhalants and pollutants chemical agents tobacco alcohol (oh no) smoke overuse of the voice inhalation of volatile gases (like when Petunia eats egg whites for breakfast) intubation GERD

If a TB test is positive, what would be the next step for the patient

Chest X-Ray

What drug is used in the treatment of Anthrax?

Ciprofloxacin 500 mg orally twice daily or 400 mg IV q 12 h

Pathophysiology of Common Clinical Manifestation of Pneumonia

Clinical Manifestation Pathophysiology CM=Increased respiratory rate/dyspnea P=Stimulation of chemoreceptors Increased work of breathing as a result of decreased lung compliance Stimulation of J receptors Anxiety Pain CM=Hypoxemia P=Alveolar consolidation Pulmonary capillary shunting CM=Cough P=Fluid accumulation in the receptors of the trachea, bronchi, and bronchioles CM=Purulent, blood-tinged, or rust-colored sputum P=A result of the inflammatory process in which fluid from the pulmonary capillaries and red blood cells moves into the alveoli CM=Fever P=Phagocytes release pyrogens that cause the hypothalamus to increase body temperature CM=Pleuritic chest discomfort P=Inflammation of the parietal pleura causes pain on inspiration

Drug therapy for TB

Combination drug therapy with strict adherence: (All are first line meds): Isoniazid (INH) Rifampin (RIF) Pyrazinamide Ethambutol With INH and RIF - watch for liver damage Three negative sputum cultures = no longer infectious

Ethambutol

Consult the physician for dose reduction when the pt complains of blurry vision after starting treatment Avoid drinking alcoholic beverages Teach patients to identify changes in the ability to differentiate colors.

Which components belong to the ventilator bundle approach to prevent ventilator-associated pneumonia (VAP)? Select all that apply. A. Administering antibiotic prophylaxis B. Continuous removal of subglottic secretions C. Elevating the head of the bed at least 30 degrees whenever possible D. Handwashing before and after contact with the client E. Placing a nasogastric tube F. Placing the client in a negative airflow room

Continuous removal of subglottic secretions Elevating the head of the bed at least 30 degrees whenever possible Handwashing before and after contact with the client

c. Respiratory infection

Deteriorating cilia in the respiratory tract predispose the elderly to which of the following? a. Chronic hypoxia b. Pulmonary hypertension c. Respiratory infection d. Decreased ventilation

-Vertigo -Hypertension -Insomnia

Drug therapy with antihistamines & decongestants to treat sinusitis are used with caution in the older adult b/c of which possible side effects?

a. Diaphragm moves down

During inhalation, which of the following muscle con- tractions takes place to enlarge the chest cavity from top to bottom? a. Diaphragm moves down. b. External intercostal muscles move down. c. Diaphragm moves up. d. Internal intercostal muscles move up

Ventilator bundle

Elevate HOB at least 30 degrees, hand hygiene, constant oral care

The oxygen saturation monitor of the client recovering from an empyema indicates periodic decreased perfusion, yet the client is talking and laughing with a visitor. The client's respirations are even and unlabored, and the nail beds are pink. What does the nurse do first? A. Auscultates breath sounds B. Calls Respiratory Therapy C. Ensures that the pulse oximetry probe is in place D. Instructs the client to breathe deeply

Ensures that the pulse oximetry probe is in place

Acute Viral and Bacterial Pharyngitis differrences

Feature Viral Pharyngitis = V Bacterial Pharyngitis = B Temperature V=Low-grade or no fever B=High temperature (>101° F [38° C], and usually 102°-104° F [39°-40° C])* Ear manifestations V=Retracted or dull tympanic membrane B=Retracted or dull tympanic membrane Throat manifestations V=Scant or no tonsillar exudate. Slight erythema of pharynx and tonsils. B=Severe hyperemia of pharyngeal mucosa, tonsils, and uvula. Erythema of tonsils with yellow exudate Neck manifestations V=Possible lymphadenopathy B=Anterior cervical lymphadenopathy and tenderness Skin manifestations V=No rash B=Possible scarlatiniform rash Possible petechiae on chest or abdomen or both Dysphagia, odynophagia V=Present B=Present Other symptoms V=No cough, Rhinitis, Mild Hoarseness, Headache B=No cough, Voice characterized by pain on voicing and slurred speech, headache, arthralgia, myalgia characterized by pain on voicing and slurred speech Laboratory data V=Complete blood count usually normal White blood cell count usually ≤10,000/mm3 Negative throat culture results B=Complete blood count abnormal White blood count abnormal Throat culture results positive for beta-hemolytic streptoccus Onset V=Gradual B=Abrupt

Tonsillitis interventions nonsurgical

Finish your antibiotics rest increase fluid intake humidify the air use analgesics for pain gargle several X's each day with warm saline use throat lozenges containing mild analgesics

SARS

From "coronaviruses" family Virus infection of respiratory tract cells, triggering inflammatory response No known effective treatment Must prevent spread of infection Strict airborne isolation Handwashing

Lab assessment pneumonia

Gram stain, culture and sensitivity of sputum CBC ABGs BUN Electrolytes Creatinine Maybe an HIV test Urinalysis for blood, pus or protein which may occur in the septic patient

Seasonal Flu

Highly contagious acute viral respiratory infection 5% to 20% of population per year >36,000 deaths per year Viruses: A, B, C Type C flu symptoms are much less severe. Vaccination is widely available Antiviral agents may be effective

d. Patient has a 25 pack-year smoking history

How should the nurse record smoking history on a patient who has smoked 2.5 packs of cigarettes per day for 10 years? a. Patient has smoked cigarettes for 10 years. b. Patient smokes 2.5 packs of cigarettes per day. c. Patient has a 12.5 pack-year smoking history. d. Patient has a 25 pack-year smoking history

Primary nursing Dx for pneumonia

Impaired Gas Exchange r/t effects of alveolar-capillary membrane changes Ineffective Airway Clearance r/t effects of infection, excessive tracheobronchial secretions,fatigue and decreased energy, chest discomfort, muscle weakness.

What are some important assessment findings regarding pulmonary empyema?

Important history findings include recent febrile illness (including pneumonia), chest pain, dyspnea, cough, and trauma. Observe and document the character of the sputum. Chest wall motion may be reduced on physical examination. If a pleural effusion is present, fremitus may be decreased or absent on palpation, percussion may sound flat, and breath sounds are decreased on auscultation. With compression of lung tissue near the effusion, abnormal breath sounds include bronchial breath sounds, egophony, and whispered pectoriloquy. Ask about fever, chills, night sweats, and weight loss. The patient may be hypotensive because of a mediastinal deviation placing pressure on the heart. Palpate the point of maximal impulse (PMI) because it may be displaced.

Prevent the spread of infection to other employees & patients

In the event on a new SARS outbreak, what's the nurse's primary role?

Venilator associated pneumonia VAP

Incidence increasing, especially with ET tubes in place for mechanical ventilation "Ventilator bundles" (care to reduce incidence) Hand hygiene Oral care Head of bed elevation

What is laryngitis

Inflammation of mucous membranes lining the larynx; possible edema of vocal cords Recurrent, may be caused by GERD

Sinusitis disease process

Inflammation of sinus mucous membranes Usually caused by Streptococcus pneumoniae, Haemophilus influenzae, Diplococcus, Bacteroides

Tonsillitis

Inflammation/infection of tonsils and lymphatic tissues Acute or chronic Contagious airborne infection, usually bacterial Antibiotics for 7 to 10 days Surgical intervention - after infection is cleared Key features - chart 33-3

What are the stages of inhalation anthrax illness?

Inhalation anthrax is a two-stage illness—prodromal and fulminant. The prodromal stage is early and difficult to distinguish from influenza or pneumonia. Manifestations include fever, fatigue, mild chest pain, and a dry, harsh cough. A special feature of inhalation anthrax is that it is not accompanied by upper respiratory manifestations of sore throat or rhinitis. Usually the patient starts to feel better and symptoms improve in 2 to 4 days. The fulminant stage begins after the patient feels a little better. Usually there is a sudden onset of breathlessness. This sensation rapidly progresses to severe respiratory distress, dyspnea, diaphoresis, stridor, and cyanosis. The patient has a high fever. Mediastinitis and pleural effusions develop. As the disease spreads through the blood, causing septic shock and meningitis, death often occurs within 24 to 36 hours even if antibiotics are started in this stage.

What signifies a positive TB test (reaction or dormant)?

Injection site reaction of >10 mm

Sinitis Surgical management Caldwell-Luc

Inscision under the upper lip into the maxillary sinus then infected mucosa is removed.

ANS: D A common cause of bacterial pharyngitis is group A streptococcal virus, which can lead to serious complications. Both RATs and culture and sensitivity can diagnose this bacterium; however, with an RAT, the health care provider can obtain results in about 15 minutes, and definitive treatment can begin much sooner. A broad-spectrum antibiotic would not be administered before it was determined whether the infection was bacterial. A sputum specimen is needed for lung infection but not for throat infection.

It is suspected that a client has bacterial pharyngitis. What is the best intervention? a. Administer a broad-spectrum antibiotic. b. Have the client produce a sputum specimen. c. Obtain samples for culture and sensitivity. d. Assess a rapid antigen test (RAT).

What are clinical manifestations of pneumonia?

Many patients with pneumonia have flushed cheeks, bright eyes, and an anxious expression. The patient may have chest or pleuritic pain or discomfort, myalgia, headache, chills, fever, cough, tachycardia, dyspnea, tachypnea, hemoptysis, and sputum production. Severe chest muscle weakness also may be present from sustained coughing. Observe the patient's breathing pattern, position, and use of accessory muscles. The hypoxic patient may be uncomfortable in a lying position and will sit upright, balancing with the hands. Assess the cough and the amount, color, consistency, and odor of sputum produced. Crackles are heard with auscultation when fluid is in interstitial and alveolar areas. Wheezing may be heard if inflammation or exudate is in the airways. Bronchial breath sounds are heard over areas of density or consolidation. Fremitus is increased over areas of pneumonia, and percussion is dulled. Chest expansion may be diminished or unequal on inspiration. In evaluating vital signs, compare the results with baseline values. The patient with pneumonia is likely to be hypotensive with orthostatic changes as a result of vasodilation and dehydration, especially the older adult. A rapid, weak pulse may indicate hypoxemia, dehydration, or impending shock. Dysrhythmias may be present as a result of cardiac tissue hypoxia.

Pandemic Flu

Mostly prevalent among animals and birds; virus can mutate becoming infectious to humans Examples: H1N1 (swine flu), H5N1 (bird flu) Strict isolation precautions! Pandemic flu is considered a disaster!

Allergic rhinitis

Often called hay fever or allergies, is triggered by hypersensitivity reactions to airborne allergens, especially plant pollens or molds. Some episodes are "seasonal" because they recur at the same time each year and last a few weeks.

S&S of COPD

Orthopneic, use of accessory muscles to breathe, thin appearance, barrel chest, digital clubbing, chronic cough, wheezing, pursed-lip breathing, increased sputum, dyspnea, prolonged expiratory time, easily fatigued, frequent respiratory infections----->>late s&s: cor pulmonale (right sided heart failure)

Interventions for pneumonia

Oxygen Therapy: Administration of oxygen and monitoring of its effectiveness •Clear oral, nasal, and tracheal secretions, as appropriate. •Restrict smoking. •Maintain airway patency. •Set up oxygen equipment and administer through a heated, humidified system. •Monitor the oxygen liter flow. •Monitor position of oxygen delivery device. •Instruct patient about importance of leaving oxygen delivery device on. •Periodically check oxygen delivery device to ensure that the prescribed concentration is being delivered. •Monitor the effectiveness of oxygen therapy (e.g., pulse oximetry, ABGs), as appropriate. •Assure replacement of oxygen mask/cannula whenever the device is removed. •Monitor patient's ability to tolerate removal of oxygen while eating. •Change oxygen delivery device from mask to nasal prongs during meals, as tolerated. •Monitor for signs of oxygen toxicity and absorption atelectasis. •Monitor patient's anxiety related to need for oxygen therapy. •Monitor for skin breakdown from friction of oxygen device. •Provide for oxygen when patient is transported. ABG, Arterial blood gas.

Analysis & Planning & Implementation of Pneumonia

Priority problems for the patient with pneumonia are: 1. Hypoxemia related to decreased diffusion at the alveolar-capillary membrane 2. Potential for airway obstruction related to excessive tracheobronchial secretions, fatigue, chest discomfort, muscle weakness 3. Potential for sepsis related to the presence of microorganisms in a very vascular area -Nursing priorities include delivery of oxygen therapy & assisting the patient with bronchial hygiene. -Oxygen therapy is usually delivered by nasal cannula or mask unless the hypoxemia doesn't improve with these devices. The patient who is confused may not tolerate a facemask. -Incentive spirometry is a type of bronchial hygiene used in pneumonia. Objective is to improve inspiratory muscle action & to prevent or reverse atelectasis (alveolar collapse). Instruct the patient to exhale fully, then place the mouthpiece in his or her mouth, & then take a long, slow, deep breath for 3 to 5 seconds. *PREVENTING AIRWAY OBSTRUCTION** -Interventions to avoid airway obstruction in pneumonia are similar to those for COPD or asthma. Because of fatigue, muscle weakness, chest discomfort, & excessive secretions, the patient often has difficulty clearing secretions. -Encourage the alert patient to drink at least 2 liters of fluid daily to prevent dehydration unless another health problem requires fluid restriction. -Bronchodilators, especially beta2 agonists, are prescribed when bronchospasm is present. -Initially given by nebulizer & then by metered-dose inhaler. -Inhaled or IV steroids are used with acute pneumonia when airway swelling is present. **PREVENTING SEPSIS** -Key to effective treatment of pneumonia is eradication of the organism causing the infection. -When sepsis accompanies pneumonia, the risk for death is high. -For pneumonia resulting from aspiration of food or stomach contents, interventions focus on preventing lung damage & treating the infection. -Aspiration of acidic stomach contents can cause widespread inflammation, leading to acute respiratory distress syndrome (ARDS) & permanent lung damage. In these conditions, steroids & NSAIDs are used with antibiotics to reduce the inflammatory response.

Sinusitis s/p interventions

S/P Sinus surgery: Semi Fowler's Gentle oral hygiene Ice compresses for 24 hours Change "mustache" dressing and record the type of drainage Teach the family to change the dressing Soft foods/ increase fluids Recliner or pillows to keep the head at 20 degrees Humidifier at night Limit Valsalva maneuver (no coughing, blowing the nose, or straining at stool) for at least 2 weeks postoperatively to prevent bleeding and tissue damage. Teach the patient to take his or her temperature twice daily during the first week after surgery and to report an elevation to 100° F or higher to the surgeon.

S/S of flu

Severe headache, muscle ache, fever, chills, fatigue, weakness, anorexia

Combination drug therapy is effective in preventing transmission. Combination drug therapy is the most effective method of treating tuberculosis (TB). Multiple drug regimens destroy organisms as quickly as possible. The use of multiple drugs reduces the emergence of drug-resistant organisms.

The client is homeless and has been living in shelters for the past 6 months. The client asks the nurse why he must take so many medications. What information will the nurse provide in answering this question? Select all that apply. A. Combination drug therapy is effective in preventing transmission. B. Combination drug therapy is the most effective method of treating tuberculosis (TB). C. Combination drug therapy will decrease the length of required treatment to 2 months. D. Multiple drug regimens destroy organisms as quickly as possible. E. The use of multiple drugs reduces the emergence of drug-resistant organisms.

ANS: B Decreased lung sounds and decreased lung expansion could indicate the development of a complication such as empyema or pus in the pleural space. The nurse should check the client's oxygen saturation and notify the provider. Infection can also move into the bloodstream and result in sepsis, so quick treatment is needed.

The nurse assesses a client with pneumonia and notes decreased lung sounds on the left side and decreased lung expansion. What is the nurse's best action? a. Have the client cough and deep breathe. b. Check oxygen saturation and notify the health care provider. c. Perform an arterial blood gas analysis. d. Increase oxygen flow to 10 L/min.

ANS: A A dry cough and difficulty swallowing may indicate that the client is developing laryngitis. The nurse should assess whether the client can speak or shows any changes in his or her voice. The other interventions are not appropriate.

The nurse has determined that a client has an acute sore throat. What is the nurse's best action? a. Assess whether the client can speak. b. Call an ear-nose-throat specialist. c. Administer an antibiotic. d. Give the client ice chips.

Scheduled & PRN aerosol nebulizer bronchodilator treatments

The nurse has identified the priority patient problem of Ineffective Airway Clearance with bronchospasms for a patient with pneumonia. The pt has no previous history of chronic respiratory disorders. The nurse obtains an order for which nursing intervention?

"The intranasal vaccine can be given to everybody in the family."

The nurse has taught the client about influenza infection control. Which client statement indicates the need for further teaching? A. "Handwashing is the best way to prevent transmission." B. "I should avoid kissing and shaking hands." C. "It is best to cough and sneeze into my upper sleeve." D. "The intranasal vaccine can be given to everybody in the family."

ANS: C, D, E The client should not stop the drug merely because he or she has no manifestations. The client will need to be on the drug for longer than 1 month. The nurse should teach the client about long-term antibiotic therapy to help with compliance. Inhalation anthrax is not spread by person-to-person contact, so isolation would not be necessary. The client would not need a mask. Health care providers need only use handwashing and Standard Precautions. Always report inhalation anthrax to authorities because it is considered an intentional act of terrorism.

The nurse is caring for a client who has inhalation anthrax. What nursing actions are of the highest priority? (Select all that apply.) a. Placing the client in an isolation room b. Teaching the client how to use a mask c. Teaching the client about long-term antibiotic therapy d. Using handwashing and other Standard Precautions e. Reporting suspected cases to the proper authorities

ANS: B, D, F The nurse should follow Airborne Precautions when caring for clients suspected of SARS. Wear a gown and goggles when in the room and caring for the client. Use a disposable particulate mask respirator if the client is coughing, or if particles are being aerosolized. Handwashing and Standard Precautions are not enough. The client does not have to wear a mask while others are in the room because they should be protecting themselves by using Airborne Precautions.

The nurse is caring for a client who is suspected of having severe acute respiratory syndrome (SARS). What actions by the nurse are most appropriate? (Select all that apply.) a. Wash hands when entering the client's room and use Standard Precautions. b. Wear a gown and goggles when entering the client's room. c. Teach the client to wear a mask at all times when someone is in the room. d. Use a disposable particulate mask respirator when the client is coughing. e. Keep the door to the client's room open to allow close monitoring. f. Place the client in a negative airflow room, if available in the facility.

ANS: C Increasing fluids has been proven to decrease the thickness of secretions, thus allowing them to be expectorated quickly. The other interventions would not be as effective.

The nurse is teaching a client with pneumonia ways to clear secretions. Which intervention is the most effective? a. Administering an antitussive medication b. Administering an antiemetic medication c. Increasing fluids to 2 L/day if tolerated d. Having the client cough and deep breathe hourly

Stridor Drooling

The nurse is teaching the pt & family about care of a peritonsillar abscess at home. For what symptoms does the nurse indicate the need for the pt to go to the ED immediately?

What are common manifestations of SARS?

Usually, the patient has a fever higher than 100.4° F (38.0° C), a headache, and general body aches. Mild cold symptoms of a runny nose, sore throat, and watery eyes may also be present. Within 2 to 7 days, the patient develops a dry cough and has difficulty breathing. Hypoxia, with cyanosis, low oxygen saturation, and a feeling of breathlessness, indicates more severe illness. Chest x-rays show a pattern similar to pneumonia. Diagnosis is made by the manifestations and the use of a rapid SARS test that detects SARS-CoV RNA in the blood within 2 days after symptoms begin. -No known effective treatment for this infection exists at this time. Standard antibiotic agents & antiviral drugs can't kill the virus or prevent its replication. -Oxygen is given when hypoxia or breathlessness is present. Therapies to dilate the bronchioles & move respiratory secretions are used. -Isolating the person with SARS & adhering to strict Transmission Precautions are effective in containing the infection & preventing an epidemic. -A major nursing responsibility is the prevention of infection spread. Use strict airborne isolation techniques. Handwashing is a key prevention intervention. Use Airborne Precautions & Contact Precautions with patients who are suspected to have SARS. Use gown & eye protection when coming into direct contact with the patient.

Acute Viral & Bacterial Pharyngitis

Viral- temp: low grade or no fever -ears- retracted or dull tympanic membrane throat- scant or no tonsillar exudate; slight erythema of pharynx & tonsils Neck- possible lymphadenopathy -Skin- no rash -Dysphagia/odynophagia= present -Other symptoms= no cough; rhinitis; mild hoarseness; headache -Lab data: CBC- usually normal; WBC usually < 10,000; negative throat culture results Onset: Gradual -Bacterial -Temp: High temp usually 102-104 Ear: retracted or dull tympanic membrane -Throat- severe hyperemia of pharyngeal mucosa, tonsils & uvula; erythema of tonsils with yellow exudates -Neck: Anterior cervical lymphadenopthy & tenderness -Skin- possible scarlatiniform; possible petechiae on chest or abdomen or both Dysphagia;odynophagia: Present -Other symptoms: No cough; voice characterized by pain on voicing & slurred speech; headache; arthralgia; myalgia LAB-Data: CBC- abnormal; WBC usually >12,000; throat culture results positive for beta hemolytic Streptococcus; Onset: Abrupt

ANS: B, C, D Providing frequent oral care, keeping the head of the bed elevated, and maintaining good hand hygiene are currently stated as the best ways to help prevent VAP. Prophylactic antibiotics are not recommended; neither is taking the client off the ventilator. Likewise, frequent chest percussion is not stated as an intervention to decrease VAP.

What is the best way for the nurse to decrease the risk of ventilator-associated pneumonia (VAP) in a ventilator-dependent client? (Select all that apply.) a. Provide prophylactic antibiotics. b. Provide frequent oral care. c. Keep the head of the bed elevated. d. Maintain good hand hygiene. e. Perform chest percussion frequently.

ANS: C The client with SARS can rapidly develop hypoxia. Assessing oxygenation is a priority because intubation and mechanical ventilation may be needed. Maintaining precautions is essential for preventing the spread of this illness, but oxygenation and client safety are the highest priorities. Antibiotics are administered if bacterial pneumonia occurs with this disease. Hydration is important to make sure secretions stay liquefied; this is also secondary to oxygenation.

What is the priority nursing intervention when caring for a client with severe acute respiratory syndrome (SARS)? a. Maintaining Standard Precautions b. Administering antibiotics c. Assessing oxygenation d. Making sure the client stays hydrated

Continuous tube feedings Bronchoscopy procedure Decreased level of consciousness Stroke

Which conditions may cause patients to be at risk for aspiration pneumonia?

Complete blood count

Which diagnostic test is NOT used to diagnose TB? (chest radiography, mantoux skin test, & sputum culture ARE used)

Sputum gram stain Chest x-ray

Which diagnostic tests are most likely to be done for a patient suspected of having community acquired pneumonia?

Viruses Irritants Bacteria Alcohol

Which factors can contribute to acute pharyngitis?

b. "Exhale, place canister in mouth, depress canister and inhale at the same time."

Which instruction is correct when teaching a patient how to use a metered-dose inhaler? a. "Inhale deeply, place canister in mouth, depress top of canister, exhale." b. "Exhale, place canister in mouth, depress canister and inhale at the same time." c. "Cough, place canister in mouth, inhale deeply, cough again." d. "Exhale, depress canister, place in mouth, inhale deeply."

ANS: D Nicotine constricts blood vessels, increases mean arterial pressure, and increases afterload. Smoking while using a nicotine patch increases afterload to such an extent that the myocardium must work harder (with the coronary arteries constricted) and may cause a myocardial infarction. Abruptly discontinuing the patch will not necessarily cause hypertension or nausea and vomiting. Smoking while using the patch will not increase the risk for pneumonia. pg 652

Which is a priority teaching intervention for the client who is using a nicotine patch? a. "Abruptly discontinuing this patch can cause high blood pressure." b. "Abruptly discontinuing this patch can cause nausea and vomiting." c. "Smoking while using this patch increases the risk for pneumonia." d. "Smoking while using this patch increases the risk for a heart attack."

Vital capacity

amt of air displaced by maximum exhalation

TB is transmitted via aerosolization. Aerosolization means...

an airborne route

Chronic Bronchitis

an inflammation of the bronchi and bronchioles caused by exposure to irritants, especially cigarette smoke; triggers inflammation, vasodilation, mucosal edema, congestion, and bronchospasm; affects only the airway not the alveoli. Chronic inflammation---->> increases the number of mucous glands---->> increased production of thick mucous--->> thickened bronchial walls and mucous plugs impair flow and narrow airway

What is the most effective method of treating TB?

combination drug therapy

Polycythemia

compensatory increase in RBCs in the chronically hypoxic patient

SARS manifestions

fever higher than 100.4 headache general body aches mild cold Sx Within 2-7 days dry cough, difficulty breathing Late stage: hypoxia, cyanosis, low o2 stats, feeling of breathlessness Chest xray looks like pneumonia Dx with SARS test: in the blood after 2 days when Sx start

Environmental causes of pneumonia

inhalation of toxic gases, chemicals and smoke aspiration of water, food, fluid and vomitus

Pharyngitis

or "sore throat" is a common inflammation of the pharyngeal mucous membranes that often occurs with rhinitis & sinusitis. -Acute pharyngitis- can be caused by bacteria, viruses, other organisms, trauma, dehydration, irritants, & tobacco or alcohol use. -Throat soreness & dryness, throat pain, pain on swallowing (odynophagia), difficulty swallowing, & fever. Mild to severe redness may be seen with or without enlarged tonsils & with or without exudate. Nasal discharge- which varies from thin & watery to thick & purulent. Enlargement of neck lymph nodes occurs with both viral & bacterial. -Bacterial often associated with enlarged red tonsils, exudate, purulent nasal discharge, & local lymph node enlargement.

What makes the mortality rate even higher with VAP

severe hypoxemia (arterial o2 < 80 mm hg) those who have atelectasis, pleural effusion or ventilator failure

Risk factors for HAP

• Is an older adult • Has a chronic lung disease • Has presence of gram-negative colonization of the mouth, throat, and stomach • Has an altered level of consciousness • Has had a recent aspiration event • Has presence of endotracheal, tracheostomy, or tubenasogastric • Has poor nutritional status • Has immunocompromised status (from disease or drug therapy) • Uses drugs that increase gastric pH (histamine [H2] blockers, antacids) or alkaline tube feedings • Is currently receiving mechanical ventilation (ventilator- acquired pneumonia [VAP])

Risk factors for community acquired pneumonia

• Is an older adult • Has never received the pneumococcal vaccination or received it more than 6 years ago • Did not receive the influenza vaccine in the previous year • Has a chronic health problem or other coexisting condition • Has recently been exposed to respiratory viral or influenza infections • Uses tobacco or alcohol

Key Features of Acute Tonsillitis

• Sudden onset of a mild to severe sore throat • Fever • Muscle aches • Chills • Dysphagia, odynophagia (painful swallowing of food) • Pain in the ears • Headache • Anorexia • Malaise • "Hot potato" voice (thickened voice of poor quality) • Tonsils visually swollen and red with pus • Tonsils may be covered with a white or yellow exudate • Purulent drainage may be expressed by pressing a tonsil • Uvula visually edematous or inflamed • Cervical lymph nodes usually tender and enlarged


Ensembles d'études connexes

Physics chapter 2 The structure of matter

View Set

Anatomy and Physiology Chapter 1

View Set

Human Resource Management Test #1

View Set

CIRCLES area and circumference in terms of PI

View Set

Ch 13 Labor and Birth Process 30Qw/exp

View Set

Chapter 8 environmental health & toxicology

View Set

Cell structure and function study guide

View Set

Speaking and Listening: Planning a Multimedia Presentation Instruction (instruction & assignment)

View Set